physio exam 4 Flashcards

1
Q

Transportation functions of the circulatory system include ____.

A. Carrying oxygen and carbon dioxide
B. Carrying leukocytes
C. Carrying absorbed products of digestion
D. Carrying metabolic waste
E. All of the choices are correct

A

E. All of the choices are correct

How well did you know this?
1
Not at all
2
3
4
5
Perfectly
2
Q

How much blood does the average-sized adult have?

A. 7 liters
B. 4 liters
C. 5 liters
D. 2 liters

A

C. 5 liters

How well did you know this?
1
Not at all
2
3
4
5
Perfectly
3
Q

The development of ____ would be inhibited if the actions of granulocyte colony-stimulating factor were blocked.

A. Neutrophils
B. Thrombocytes
C. Erythrocytes
D. lymphocytes

A

A. Neutrophils

How well did you know this?
1
Not at all
2
3
4
5
Perfectly
4
Q

An elite marathon runner would likely benefit from increased oxygen carrying capacity in the blood from the use of ____, which would serve to ____.

A. Erythropoietin; increase red blood cell formation

B. Granulocyte colony-stimulating factor; erythrocyte production

C. Thrombopoietin; increase megakaryocyte production

D. Hepcidin; increase leukopoiesis

A

A. Erythropoietin; increase red blood cell formation

How well did you know this?
1
Not at all
2
3
4
5
Perfectly
5
Q

A patient has a large portion of his stomach removed during a weight reduction surgery. Following this procedure, he experiences fatigue and is short of breath upon exertion. His surgery most likely resulted in ____.

A. Iron-deficiency anemia
B. Leukemia
C. Pernicious anemia
D. Aplastic anemia

A

C. Pernicious anemia

How well did you know this?
1
Not at all
2
3
4
5
Perfectly
6
Q

What is the enzyme that converts soluble fibrinogen to insoluble fibrin?

A. Plasmin
B. Thrombin
C. Prothrombin
D. thromboplastin

A

B. Thrombin

How well did you know this?
1
Not at all
2
3
4
5
Perfectly
7
Q

The prostaglandin, thromboxane A2, stimulates ____.

A. Platelet repelling
B. Platelet aggregation
C. Vasoconstriction
D. Vasodilation

A

B. Platelet aggregation

How well did you know this?
1
Not at all
2
3
4
5
Perfectly
8
Q

What type of antigens are present in Type O blood?

A. A only
B. B only
C. Both A and B
D. Neither A nor B

A

D. Neither A nor B

How well did you know this?
1
Not at all
2
3
4
5
Perfectly
9
Q

The most abundant leukocytes in the blood are the ____.

A. Neutrophils
B. Monocytes
C. Basophils
D. eosinophils

A

A. Neutrophils

How well did you know this?
1
Not at all
2
3
4
5
Perfectly
10
Q

Erythroblastosis fetalis occurs when _____.

A. The mother has Rh- blood and the fetus has Rh+ blood

B. The mother has type AB blood and the fetus has type O blood

C. The mother has Rh+ blood and the fetus has Rh- blood

D. The mother has blood type A and the fetus has blood type O

A

A. The mother has Rh- blood and the fetus has Rh+ blood

How well did you know this?
1
Not at all
2
3
4
5
Perfectly
11
Q

The ____ of the heart are involved with ____.

A. Left side; circulating blood high in carbon dioxide and low in oxygen

B. Atria; receiving deoxygenated blood

C. Atria, ejecting blood from the heart

D. Ventricles; moving blood into major arteries

A

D. Ventricles; moving blood into major arteries

How well did you know this?
1
Not at all
2
3
4
5
Perfectly
12
Q

In the fetus, blood can flow from the right atrium to the left atrium through the ____.

A. Ductus arteriosus
B. Tricuspid valve
C. Ductus venosus
D. Foramen ovale

A

D. Foramen ovale

How well did you know this?
1
Not at all
2
3
4
5
Perfectly
13
Q

What causes the semilunar valves to close?

A. Higher pressure in the aorta and pulmonary trunk than in the ventricles

B. Higher pressure in the atria than in the ventricles

C. Higher pressure in the ventricles than aorta and pulmonary trunk

D. Higher pressure in the ventricles than in the atria

A

A. Higher pressure in the aorta and pulmonary trunk than in the ventricles

How well did you know this?
1
Not at all
2
3
4
5
Perfectly
14
Q

Cardiac ____ channels are called hyperpolarization cyclic nucleotide (HCN) channels because they open in response to hyperpolarization.

A. Purkinje
B. Atrial
C. Pacemaker
D. ventricular

A

C. Pacemaker

How well did you know this?
1
Not at all
2
3
4
5
Perfectly
15
Q

The production and conduction of action potentials in the heart produces a recordable tracing of the electrical activity of the heart called a(n) ____.

A. Depolarization
B. EMG
C. ECG
D. Heart scan

A

C. ECG

How well did you know this?
1
Not at all
2
3
4
5
Perfectly
16
Q

Most of the circulating blood can be found in ____.

A. Arteries
B. Veins
C. Capillaries
D. arterioles

A

B. Veins

How well did you know this?
1
Not at all
2
3
4
5
Perfectly
17
Q

Within the blood vessels, valves are found in the ____.

A. Capillaries
B. Arteries
C. Arterioles
D. veins

A

D. veins

How well did you know this?
1
Not at all
2
3
4
5
Perfectly
18
Q

Which type of an AV node block occurs when no atrial waves can pass through the AV node?

A. Third-degree
B. Fourth-degree
C. Second-degree
D. first-degree

A

A. Third-degree

How well did you know this?
1
Not at all
2
3
4
5
Perfectly
19
Q

In order to be effective against high cholesterol, statins must ____.

A. Inhibit HMG-coenzyme A reductase
B. Stimulate proconvertin
C. Inhibit streptokinase
D. All of the choices are correct

A

A. Inhibit HMG-coenzyme A reductase

How well did you know this?
1
Not at all
2
3
4
5
Perfectly
20
Q

Which of the following is a difference between veins and lymphatic vessels?

A. Interstitial fluid only moves into lymphatic vessels

B. Only veins contain valves

C. Only fluid in veins moves by waves of peristaltic contraction

D. Veins serve as a reservoir for the majority of blood in circulation

A

D. Veins serve as a reservoir for the majority of blood in circulation

How well did you know this?
1
Not at all
2
3
4
5
Perfectly
21
Q

Damage to tissues stimulates the activation of the ________ pathway.

A. complement
B. hemophilic
C. extrinsic
D. intrinsic

A

C. extrinsic

How well did you know this?
1
Not at all
2
3
4
5
Perfectly
22
Q

Both the intrinsic and extrinsic clotting pathways activate factors that ________.

A. convert prothrombin to thrombin
B. convert fibrinogen to fibrin
C. convert plasminogen to plasmin
D. None of the choices are correct

A

A. convert prothrombin to thrombin

How well did you know this?
1
Not at all
2
3
4
5
Perfectly
23
Q

Aspirin inhibits blood clotting by directly preventing ________.

A. thrombin formation
B. fibrin formation
C. clot retraction
D. platelet plug formation

A

D. platelet plug formation

How well did you know this?
1
Not at all
2
3
4
5
Perfectly
24
Q

The major plasma protein is ________.

A. alpha globulin
B. albumin
C. beta globulin
D. fibrinogen

A

B. albumin

How well did you know this?
1
Not at all
2
3
4
5
Perfectly
25
Q

What ion is necessary for the clotting process?

A. K+
B. Mg2+
C. Ca2+
D. Na+

A

C. Ca2+

How well did you know this?
1
Not at all
2
3
4
5
Perfectly
26
Q

When the wrong blood type is given to a patient, the antibodies in the patient’s blood react with the antigens on the surface of the transfused blood causing a reaction called ________.

A. agglutination
B. neutralization
C. coagulation
D. precipitation

A

A. agglutination

How well did you know this?
1
Not at all
2
3
4
5
Perfectly
27
Q

Heart murmurs may be caused by ________. This causes valves to be less compliant, creating difficulty for blood to move from one chamber to another.

A. regurgitation
B. septal defects
C. fibrillation
D. stenosis

A

D. stenosis

How well did you know this?
1
Not at all
2
3
4
5
Perfectly
28
Q

Systole refers to the ________ of the ventricles, when pressure is ________.

A. blood flow; lowest
B. filling; highest
C. relaxation; lowest
D. contraction; highest

A

D. contraction; highest

How well did you know this?
1
Not at all
2
3
4
5
Perfectly
29
Q

Ca2+ release channels in the sarcoplasmic reticulum of the heart are opened by an increase in Ca2+ in the cytoplasm. This is referred to as _____.

A. calcium-stimulated calcium release
B. calcium-induced fibrillation
C. calcium-induced repolarization
D. calcium-stimulated depolarization

A

A. calcium-stimulated calcium release

How well did you know this?
1
Not at all
2
3
4
5
Perfectly
30
Q

The ________ conducts impulses from the AV node to the bundle branches, which lead to the Purkinje fibers.

A. ventricular septum
B. AV valve
C. AV bundle
D. SA node

A

C. AV bundle

How well did you know this?
1
Not at all
2
3
4
5
Perfectly
31
Q

Which of the following is NOT one of the three coats that comprise the walls of arteries and veins?

A. Tunica media
B. Tunica externa
C. Tunica superficia
D. Tunica interna

A

C. Tunica superficia

How well did you know this?
1
Not at all
2
3
4
5
Perfectly
32
Q

The first anatomical change associated with atherosclerosis is the appearance of ________.

A. blood clotsinterstitial fluid
B. fibroblasts
C. monocytes
D. fatty streaks

A

D. fatty streaks

How well did you know this?
1
Not at all
2
3
4
5
Perfectly
33
Q

Which of the following is NOT a lymphatic organ?

A. Thymus
B. Spleen
C. Lymph nodes
D. Tonsils

A

C. Lymph nodes

How well did you know this?
1
Not at all
2
3
4
5
Perfectly
34
Q

Interstitial fluid is derived from ________.

A. formed elements
B. cells
C. lymph
D. plasma

A

D. plasma

How well did you know this?
1
Not at all
2
3
4
5
Perfectly
35
Q

Which substance does NOT inhibit platelet aggregation?

A. Prostacyclin
B. CD39
C. von Willebrand factor
D. Nitric oxide

A

C. von Willebrand factor

How well did you know this?
1
Not at all
2
3
4
5
Perfectly
36
Q

The presence of both type-B antibodies and type-B antigens are to agglutination as ________ is to ________.

A. vascular damage; hemostasis
B. hemostasis; clotting
C. coagulation; hemophilia
D. fibrin; fibrinogen

A

A. vascular damage; hemostasis

How well did you know this?
1
Not at all
2
3
4
5
Perfectly
37
Q

Decreased liver protein synthesis would cause the amount of ________ and ________ in the blood to be decreased.

A. RBCs; WBCs
B. thromboplastin; RBCs
C. albumin; fibrinogen
D. platelets; albumin

A

C. albumin; fibrinogen

How well did you know this?
1
Not at all
2
3
4
5
Perfectly
38
Q

Which of the following stimulates the formation of the platelet plug?
A. von Willebrand factor
B. ADP
C. Thromboxane A2
D. All of the choices are correct

A

D. All of the choices are correct

How well did you know this?
1
Not at all
2
3
4
5
Perfectly
39
Q

How does hepcidin function in iron homeostasis?

A. Promotes production and insertion of ferroportin channels in enterocytes

B. Promotes cellular storage of iron by removing ferroportin channels in enterocytes

C. Increases production of transferrin

D. Increases macrophage destruction of old red blood cells to release iron

A

B. Promotes cellular storage of iron by removing ferroportin channels in enterocytes

How well did you know this?
1
Not at all
2
3
4
5
Perfectly
40
Q

What is the role of serotonin in blood clotting?

A. Causes vasoconstriction of blood vessels to slow loss of blood
B. Causes platelets to become sticky
C. Converts fibrinogen to fibrin
D. Enhances the platelet release reaction

A

A. Causes vasoconstriction of blood vessels to slow loss of blood

How well did you know this?
1
Not at all
2
3
4
5
Perfectly
41
Q

John has blood type B. In an emergency, John could receive which of the following blood types?

A. AB only
B. B or O
C. O only
D. B only

A

B. B or O

How well did you know this?
1
Not at all
2
3
4
5
Perfectly
42
Q

Which part of the red blood cell can bind to oxygen?

A. Cell cytoplasm
B. Plasma membrane
C. Heme part of hemoglobin
D. Globin part of hemoglobin

A

C. Heme part of hemoglobin

How well did you know this?
1
Not at all
2
3
4
5
Perfectly
43
Q

The pathway of blood from the heart to the lungs and back to the heart is the ________ circulation.

A. systemic
B. pulmonary
C. pleural
D. cardiac

A

B. pulmonary

How well did you know this?
1
Not at all
2
3
4
5
Perfectly
44
Q

The pulmonary semilunar valve is to the right ventricle as the ________ is to the ________.

A. right atrioventricular valve; left ventricle
B. aortic valve; aorta
C. tricuspid valve; right atrium
D. mitral valve; pulmonary vein

A

C. tricuspid valve; right atrium

How well did you know this?
1
Not at all
2
3
4
5
Perfectly
45
Q

During isovolumetric ventricular relaxation ________.

A. atrial pressure is less than ventricular pressure
B. ventricular pressure is increasing
C. the atrioventricular valves are open
D. left ventricular pressure exceeds aortic pressure

A

A. atrial pressure is less than ventricular pressure

How well did you know this?
1
Not at all
2
3
4
5
Perfectly
46
Q

Bipolar limb lead three uses the ________.

A. right arm and left leg
B. left arm and right leg
C. left and right arms
D. left arm and left leg

A

D. left arm and left leg

How well did you know this?
1
Not at all
2
3
4
5
Perfectly
47
Q

Ventricular depolarization ________.
A. appears as the P wave on an ECG
B. appears as the T wave on an ECG
C. occurs prior to atrial depolarization
D. occurs after atrial depolarization

A

D. occurs after atrial depolarization

How well did you know this?
1
Not at all
2
3
4
5
Perfectly
48
Q

Varicose veins can be caused by ________.

A. compression of abdominal aorta by a fetus during pregnancy
B. stiffened venous valves
C. excessive venous congestion
D. All of the choices are correct.

A

C. excessive venous congestion

How well did you know this?
1
Not at all
2
3
4
5
Perfectly
49
Q

Choose the incorrect statement regarding blood vessels.

A. Smooth muscle surrounds capillaries.
B. Discontinuous capillaries create sinusoids.
C. Arteries contain more muscle than veins.
D. Valves are found in veins but not arteries

A

A. Smooth muscle surrounds capillaries.

How well did you know this?
1
Not at all
2
3
4
5
Perfectly
50
Q

Damage to the AV node could cause ________ on and ECG.

A. the absence of the T wave
B. multiple P waves
C. multiple T waves
D. multiple Q waves

A

B. multiple P waves

How well did you know this?
1
Not at all
2
3
4
5
Perfectly
51
Q

Functions of the lymphatic system include all of the following EXCEPT ________.

A. transport of interstitial fluid
B. transport of absorbed peptides
C. transport of absorbed fat
D. providing immunological defense

A

B. transport of absorbed peptides

How well did you know this?
1
Not at all
2
3
4
5
Perfectly
52
Q

What is the major hematopoietic organ in the fetus?

A. Spleen
B. Liver
C. Kidneys
D. Red bone marrow

A

B. Liver

How well did you know this?
1
Not at all
2
3
4
5
Perfectly
53
Q

Iron is transported in the blood by the protein ________.

A. fibrinogen
B. albumin
C. globin
D. Transferrin

A

D. Transferrin

How well did you know this?
1
Not at all
2
3
4
5
Perfectly
54
Q

A normal hematocrit for women would be ________.

A. 40%
B. 52%
C. 30%
D. 48%

A

A. 40%

How well did you know this?
1
Not at all
2
3
4
5
Perfectly
55
Q

What substances are needed to produce red blood cells?

A. Vitamin B12
B. Folic acid
C. Iron
D. All of the choices are correct.

A

D. All of the choices are correct.

How well did you know this?
1
Not at all
2
3
4
5
Perfectly
56
Q

Which anticoagulant and its action is NOT correctly matched?

A. Aspirin—inhibits prostaglandin production and platelet aggregation

B. Citrate—combines with calcium to prevent its function with clotting factors

C. Coumadin—inhibits tissue thromboplastin

D. Heparin—inhibits action of thrombin

A

C. Coumadin—inhibits tissue thromboplastin

How well did you know this?
1
Not at all
2
3
4
5
Perfectly
57
Q

If the ________ were not properly functioning, congestion in the venous system would be an expected sign.

A. pulmonary semilunar valve
B. tricuspid valve
C. bicuspid valve
D. mitral valve

A

B. tricuspid valve

How well did you know this?
1
Not at all
2
3
4
5
Perfectly
58
Q

The P wave of an ECG represents ________.

A. ventricular depolarization
B. atrial depolarization
C. ventricular repolarization
D. atrial repolarization

A

B. atrial depolarization

How well did you know this?
1
Not at all
2
3
4
5
Perfectly
59
Q

Following musculoskeletal surgery on the lower extremity, oftentimes a return to ambulation (walking) is prescribed as soon as possible to reduce edema. Why?

A. Increases muscular hypertrophy similar as with most exercise

B. Promotes fluid return via alternating pressures created by muscular contractions

C. Increases mechanical stimulation to fibroblasts and tissue repair

D. Promotes the inflammatory response and thus speeds healing

A

B. Promotes fluid return via alternating pressures created by muscular contractions

How well did you know this?
1
Not at all
2
3
4
5
Perfectly
60
Q

The lymphatic system can help cancer cells ________ since cancer cells may enter, circulate, and later exit porous lymphatic capillaries.

A. die off
B. mutate
C. revert to normal cells
D. metastasize

A

D. metastasize

How well did you know this?
1
Not at all
2
3
4
5
Perfectly
61
Q

A normal red blood cell count is in the range of ________ /mm3.

A. 4.23–5.98 million
B. 4,300–10,800
C. 130,000–400,000
D. 120–220

A

A. 4.23–5.98 million

How well did you know this?
1
Not at all
2
3
4
5
Perfectly
62
Q

Blood globulins include which of the following types?

A. Alpha globulins
B. Beta globulins
C. Gamma globulins
D. All of the choices are correct

A

D. All of the choices are correct

How well did you know this?
1
Not at all
2
3
4
5
Perfectly
63
Q

Polycythemia would be induced by _______ in the blood.

A. Decreased carbon dioxide
B. Increased oxygen
C. Increased infection
D. Decreased oxygen

A

D. Decreased oxygen

How well did you know this?
1
Not at all
2
3
4
5
Perfectly
64
Q

What structures keep the AV valves from everting under high ventricular pressure?

A. Semilunar valves
B. Papillary muscles and chordae tendineae
C. Interventricular and interatrial septa
D. Annuli fibrosis

A

B. Papillary muscles and chordae tendineae

How well did you know this?
1
Not at all
2
3
4
5
Perfectly
65
Q

Medications that are used to treat arrhythmias may use which of the following mechanisms?

A. Block β-adrenergic receptors.
B. Block fast Na+ channels.
C. Block slow Ca2+ channels.
D. All of the choices are correct

A

D. All of the choices are correct

How well did you know this?
1
Not at all
2
3
4
5
Perfectly
66
Q

Development and prevention of atherosclerosis are complicated, multifactorial processes. Which of the following would best represent desired trends in preventing or slowing the onset of atherosclerosis?

A. Increased physical activity, smoking cessation, decreased intakes dietary fat including omega-3 fatty acids

B. Decreased C-reactive protein, increased circulating LDL levels, decreased freed radicals (oxidizing agents)

C. Decreased foam cell activity, increased statin inhibition of cholesterol synthesis, increased liver LDL receptors

A

C. Decreased foam cell activity, increased statin inhibition of cholesterol synthesis, increased liver LDL receptors

How well did you know this?
1
Not at all
2
3
4
5
Perfectly
67
Q

The structure of a ________ includes a core of nonpolar triglycerides and cholesterol esters coated by proteins, phospholipids, and some free cholesterol.

A. Lipoprotein
B. prostaglandin
C. Glycoprotein
D. Nucleic acid

A

A. Lipoprotein

How well did you know this?
1
Not at all
2
3
4
5
Perfectly
68
Q

How does the circulatory system function in a protective role in the body?

A. Prevents blood loss through clotting

B. Leukocytes fight infection

C. Delivers leukocytes to areas of inflammation and infection

D. All of the choices are correct

A

D. All of the choices are correct

How well did you know this?
1
Not at all
2
3
4
5
Perfectly
69
Q

The most common type of anemia is ________.

A. Polycythemia
B. Pernicious anemia
C. Iron-deficiency anemia
D. Aplastic anemia

A

C. Iron-deficiency anemia

How well did you know this?
1
Not at all
2
3
4
5
Perfectly
70
Q

What is the stimulus for the release of erythropoietin?

A. Low RBC count
B. Decreased blood oxygen levels
C. Low amount of hemoglobin
D. All of the choices are correct

A

D. All of the choices are correct

How well did you know this?
1
Not at all
2
3
4
5
Perfectly
71
Q

Valves are said to be ________ when they do not close properly and blood leaks through them.

A. Stenotic
B. Flappy
C. Incompetent
D. Weak

A

C. Incompetent

How well did you know this?
1
Not at all
2
3
4
5
Perfectly
72
Q

In comparing the pulmonary and systemic circulations, the pulmonary circuit ________.

A. sends blood to the lungs, is involved with oxygenating blood, and involves the generation of much larger pressures

B. sends blood throughout the body, is involved with deoxygenating blood, and involves the generation of smaller pressures

C. sends blood to the lungs, is involved with oxygenating blood, and involves much lower overall resistance to flow

D. sends blood throughout the body, is involved with decreasing blood carbon dioxide levels, and involves much lower overall resistance to flow

A

C. sends blood to the lungs, is involved with oxygenating blood, and involves much lower overall resistance to flow

How well did you know this?
1
Not at all
2
3
4
5
Perfectly
73
Q

Myocardial cells exhibit a plateau phase instead of quickly repolarizing. This is due to ________.

A. inward diffusion of calcium through slow Ca2+channels

B. inward diffusion of potassium through voltage-gated K+ channels

C. inward diffusion of sodium through fast Na+channels

D. outward pumping of sodium by the Na+/K+ pump

A

A. inward diffusion of calcium through slow Ca2+channels

How well did you know this?
1
Not at all
2
3
4
5
Perfectly
74
Q

The action potential of non-pacemaker cells is due to the ________.

A. Inward diffusion of K+
B. Outward diffusion of K+
C. Inward diffusion of Ca2+
D. Inward diffusion of Na+

A

D. Inward diffusion of Na+

How well did you know this?
1
Not at all
2
3
4
5
Perfectly
75
Q

Which type of an AV node block occurs when the rate of impulse conduction through the AV node exceeds 0.20 second?

A. Third-degree
B. Fourth-degree
C. Second-degree
D. First-degree

A

D. First-degree

How well did you know this?
1
Not at all
2
3
4
5
Perfectly
76
Q

Choose the correct statement regarding atherosclerosis.

A. It is most likely an inflammatory disease.

B. Blood C-reactive protein levels are better predictors than LDL cholesterol levels.

C. Antioxidants may be used to prevent or treat it.

D. All of the choices are correct

A

D. All of the choices are correct

How well did you know this?
1
Not at all
2
3
4
5
Perfectly
77
Q

The ________ drains lymph into the left subclavian vein.

A. Left lymphatic duct
B. Subclavian duct
C. Thoracic duct
D. Common lymphatic duct

A

C. Thoracic duct

How well did you know this?
1
Not at all
2
3
4
5
Perfectly
78
Q

Mary has type AB blood. Which type of antibodies is present in her plasma?

A. Anti-A and anti-B
B. Neither anti-A nor anti-B
C.Anti-A
D.Anti-B

A

B. Neither anti-A nor anti-B

How well did you know this?
1
Not at all
2
3
4
5
Perfectly
79
Q

Vitamin K is needed for ________.

A. directly stabilizing the fibrin polymer
B. directly activating fibrinogen
C. proper clotting factor function
D. activating antithrombin III

A

C. proper clotting factor function

How well did you know this?
1
Not at all
2
3
4
5
Perfectly
80
Q

Which blood globulin is an antibody that helps fight infection?

A. Alpha globulin
B. Beta globulin
C. Gamma globulin
D. None of the choices are correct.

A

C. Gamma globulin

How well did you know this?
1
Not at all
2
3
4
5
Perfectly
81
Q

Which capillaries have wide intercellular pores that serve as a basement membrane over the capillary endothelium for increased filtration?

A. True
B. Discontinuous
C. Continuous
D. Fenestrated

A

D. Fenestrated

How well did you know this?
1
Not at all
2
3
4
5
Perfectly
82
Q

0.12 to 0.20 second is the duration of the ________ in a normal heart.

A. QRS wave
B. Q-T interval
C. diastole
D. P-R interval

A

D. P-R interval

How well did you know this?
1
Not at all
2
3
4
5
Perfectly
83
Q

The circulatory system is able to provide regulation of other body systems by circulating ________.

A. hormones
B. blood gases
C. clotting
D. immunity

A

A. hormones

How well did you know this?
1
Not at all
2
3
4
5
Perfectly
84
Q

Which of the following is NOT a characteristic of white blood cells?

A. Have nuclei and mitochondria
B. Can leave blood vessels
C. Have amoeboid movement
D. Make up the major formed element

A

D. Make up the major formed element

How well did you know this?
1
Not at all
2
3
4
5
Perfectly
85
Q

Which of the following is NOT a granulocyte?

A. Monocyte
B. Neutrophil
C. Basophil
D. Eosinophil

A

A. Monocyte

How well did you know this?
1
Not at all
2
3
4
5
Perfectly
86
Q

Failure of the SA node to depolarize would result in an ECG ________.

A. lacking a QRS complex
B. lacking a T wave
C. lacking a P wave
D. that is isoelectric

A

C. lacking a P wave

How well did you know this?
1
Not at all
2
3
4
5
Perfectly
87
Q

Ventricular fibrillation ________.

A. is not life-threatening

B. occurs when the ventricles contract in a rapid, coordinated manner

C. is induced by defibrillators

D. often occurs due to circus rhythms

A

D. often occurs due to circus rhythms

How well did you know this?
1
Not at all
2
3
4
5
Perfectly
88
Q

Which of the following could explain a gradual decrease in erythropoietin (EPO) levels ?

A. A patient in recovery from surgery with substantial blood loss

B. A patient with chronic kidney disease

C. An inactive person beginning an exercise program

D. An endurance athlete traveling from sea level to the mountains for a month of training

A

B. A patient with chronic kidney disease

How well did you know this?
1
Not at all
2
3
4
5
Perfectly
89
Q

Which of the following cells produces antibodies?

A. Basophil
B. Plasma Cell
C. Neutrophil
D. Monocyte

A

B. Plasma Cell

How well did you know this?
1
Not at all
2
3
4
5
Perfectly
90
Q

Dark-red colored blood would be found in the _______, traveling to the ________.

A. Arteries; heart
B. Capillaries; arteries
C. Veins; heart
D. Heart; veins

A

C. Veins; heart

How well did you know this?
1
Not at all
2
3
4
5
Perfectly
91
Q

The process of blood cell formations is ______.

A. Hematopoiesis
B. Polycythemia
C. Hemogenesis
D. Leukocytosis

A

A. Hematopoiesis

How well did you know this?
1
Not at all
2
3
4
5
Perfectly
92
Q

Kallikrein is responsible for _______.

A. Clot dissolution
B. Platelet plug formation
C. Serum production
D. Fibrin formation

A

A. Clot dissolution

How well did you know this?
1
Not at all
2
3
4
5
Perfectly
93
Q

What occurs when the pressure in the ventricles drops below the pressure of the atria?

A. SL valves open
B. AV valves open
C. AV valves close
D. SL valves close

A

B. AV valves open

How well did you know this?
1
Not at all
2
3
4
5
Perfectly
94
Q

In a complete AV node block, ________.

A. No signals pass into the ventricles

B. The atria will be paced by the SA node

C. The ventricles contract due to the purkinje fiber pacemaker

D. All of the choices are correct

A

D. All of the choices are correct

How well did you know this?
1
Not at all
2
3
4
5
Perfectly
95
Q

What does it mean to be a universal recipient? A person with the blood type known as the universal recipient ________.

A. expresses O agglutinogens along with A and B antibodies

B. has type-AB blood, which contains A and B agglutinins

C. lacks the ability to produce antibodies for any donor blood type

D. has type-O blood, which contains A and B antibodies

A

C. lacks the ability to produce antibodies for any donor blood type

How well did you know this?
1
Not at all
2
3
4
5
Perfectly
96
Q

The final protein that forms a blood clot is ________.

A. thromboplastin
B. thrombin
C. fibrin
D. plasmin

A

C. fibrin

How well did you know this?
1
Not at all
2
3
4
5
Perfectly
97
Q

Which organ secretes erythropoietin?

A. Liver
B. Lungs
C. Bone marrow
D. Kidneys

A

D. Kidneys

How well did you know this?
1
Not at all
2
3
4
5
Perfectly
98
Q

The amount of blood ejected by the ventricles per beat is termed the ________.

A. cardiac output
B. end-diastolic volume
C. stroke volume
D. blood pressure

A

C. stroke volume

How well did you know this?
1
Not at all
2
3
4
5
Perfectly
99
Q

The myocardium acts as a single unit called a/an ________.

A. pacemaker
B. isoelectric unit
C. functional syncytium
D. multi-unit muscle

A

C. functional syncytium

How well did you know this?
1
Not at all
2
3
4
5
Perfectly
100
Q

Prostacyclin, PGI2, prevents ________.

A. Clot retraction
B. Vasomotion
C. Platelet aggregation
D. Vasoconstriction

A

C. Platelet aggregation

How well did you know this?
1
Not at all
2
3
4
5
Perfectly
101
Q

What part of the heart’s conduction system acts as the primary pacemaker?

A. SA node
B. Purkinje fibers
C. AV node
D. Bundle of His

A

A. SA node

How well did you know this?
1
Not at all
2
3
4
5
Perfectly
102
Q

The S-T segment of an ECG represents ________.

A. the plateau phase of the myocardial action potential

B. atrial depolarization

C. ventricular depolarization

D. passage of the electrical impulse to the atrioventricular node

A

A. the plateau phase of the myocardial action potential

How well did you know this?
1
Not at all
2
3
4
5
Perfectly
103
Q

All blood vessels are lined with ________.

A. endothelial cells
B. smooth muscle
C. elastin
D. the tunica externa

A

A. endothelial cells

How well did you know this?
1
Not at all
2
3
4
5
Perfectly
104
Q

During isovolumetric ventricular contraction, ventricular pressure is ________.

A. Increasing
B. Unchanged
C. decreasing

A

A. Increasing

How well did you know this?
1
Not at all
2
3
4
5
Perfectly
105
Q

Ischemic heart disease causes ________ on the ECG.

A. no P wave
B. inverted P waves
C. no QRS complex
D. S-T depression

A

D. S-T depression

How well did you know this?
1
Not at all
2
3
4
5
Perfectly
106
Q

The possibility of an individual developing ischemic heart disease is ________.

A. increased by diets low in saturated fat
B. decreased by increasing exercise
C. decreased by diets containing 50% fat
D. unaffected by smoking cessation

A

B. decreased by increasing exercise

How well did you know this?
1
Not at all
2
3
4
5
Perfectly
107
Q

If acetylcholine binds to its receptors in the heart, there would be ____ in heart rate.

A. An increase
B. A decrease
C. No change

A

B. A decrease

How well did you know this?
1
Not at all
2
3
4
5
Perfectly
108
Q

Venous return is increased by all of the following EXCEPT ____.

A. The skcapillaryeletal muscle pump
B. Hypertension
C. Exhalation
D. Sympathetic stimulation of the veins

A

C. Exhalation

How well did you know this?
1
Not at all
2
3
4
5
Perfectly
109
Q

Cardiac output is equal to _____.

A. Stroke volume + cardiac rate

B. Stroke volume x cardiac rate

C. Stroke volume / cardiac rate

D. Stroke volume - cardiac rate

A

B. Stroke volume x cardiac rate

How well did you know this?
1
Not at all
2
3
4
5
Perfectly
110
Q

What keeps the HCN channels open?

A. ATP
B. Ca2+
C. cAMP
D. G-proteins

A

C. cAMP

How well did you know this?
1
Not at all
2
3
4
5
Perfectly
111
Q

Hypotension could be induced by excessive ____ secretion.

A. Renin
B. Aldosterone
C. Atrial natriuretic factor
D. Andidiuretic hormone

A

C. Atrial natriuretic factor

How well did you know this?
1
Not at all
2
3
4
5
Perfectly
112
Q

How much filtrate do the kidneys produce per day?

A. 110 L
B. 180 L
C. 50 L
D. 10 L

A

B. 180 L

How well did you know this?
1
Not at all
2
3
4
5
Perfectly
113
Q

Hydrostatic pressure is ____ and colloid osmotic pressure is ____ at the arterial end of a capillary.

A. Lower; lower
B. Higher; lower
C. Higher; higher
D. Lower; higher

A

B. Higher; lower

How well did you know this?
1
Not at all
2
3
4
5
Perfectly
114
Q

Vasopressin induces renal water ____.

A. Filtration
B. Secretion
C. Excretion
D. reabsorption

A

D. reabsorption

How well did you know this?
1
Not at all
2
3
4
5
Perfectly
115
Q

Sympathetic stimulation of cholinergic fibers in the arterioles of skeletal muscle will produce ____.

A. Vasodilation
B. No effect
C. vasoconstriction

A

A. Vasodilation

How well did you know this?
1
Not at all
2
3
4
5
Perfectly
116
Q

Reactive hyperemia is ____.

A. Increased blood flow due to increased metabolism

B. Increased blood flow after removal of constriction due to accumulated metabolic products

C. Increased blood flow due to sepsis

D. All of the choices are correct

A

B. Increased blood flow after removal of constriction due to accumulated metabolic products

How well did you know this?
1
Not at all
2
3
4
5
Perfectly
117
Q

During moderate exercise cerebral blood flow ____.

A. Remains unchanged
B. Increases slightly
C. Increases significantly
D. Decreases slightly

A

B. Increases slightly

How well did you know this?
1
Not at all
2
3
4
5
Perfectly
118
Q

Coronary bypass surgery is performed ____.

A. To limit myocardial ischemia
B. To decrease blood pressure
C. To correct an aneurysm
D. To limit blood flow into a damaged coronary artery

A

A. To limit myocardial ischemia

How well did you know this?
1
Not at all
2
3
4
5
Perfectly
119
Q

When the arteriovenous anastomoses are stimulated by sympathetic nerve fibers, the superficial capillary loops are ____.

A. By passed
B. Collapsed
C. Engorged with blood
D. Not affected

A

C. Engorged with blood

How well did you know this?
1
Not at all
2
3
4
5
Perfectly
120
Q

In response to increased internal temperature, blood flow to the skin will

A. Increase
B. Remain unchanged
C. decrease

A

A. Increase

How well did you know this?
1
Not at all
2
3
4
5
Perfectly
121
Q

Inhibition of the baroreceptor reflex would prevent proper regulation of all but ____.

A. Heart rate
B. Blood pressure
C. Stroke volume
D. Respiratory rate

A

D. Respiratory rate

How well did you know this?
1
Not at all
2
3
4
5
Perfectly
122
Q

A failure of blood pressure to increase upon standing is known as ____.

A. Postural tachycardia
B. Postural hypotension
C. Postural atherosclerosis
D. Postural bradycardia

A

B. Postural hypotension

How well did you know this?
1
Not at all
2
3
4
5
Perfectly
123
Q

Arterial blood pressure is influenced by ____.

A. Vasoconstriction in the peripheral arterioles

B. Increased heart rate

C. Increased stroke volume

D. All of the choices are correct

A

D. All of the choices are correct

How well did you know this?
1
Not at all
2
3
4
5
Perfectly
124
Q

Which blood vessels have the lowest pressure?

A. Venules
B. Capillaries
C. Veins
D. arterioles

A

C. Veins

How well did you know this?
1
Not at all
2
3
4
5
Perfectly
125
Q

After several tests are performed, a patient is diagnosed with left-sided heart failure. Signs and symptoms in her history and testing should include ____.

A. All of the choices are correct
B. Decreased cardiac output
C. Difficulty breathing
D. Feelings of fatigue upon exertion

A

A. All of the choices are correct

How well did you know this?
1
Not at all
2
3
4
5
Perfectly
126
Q

A patient is diagnosed with secondary hypertension. This may result from ____.

A. An epinephrine antagonist
B. An atrial natriuretic factor
C. A renin secreting tumor
D. An aldosterone antagonist

A

C. A renin secreting tumor

How well did you know this?
1
Not at all
2
3
4
5
Perfectly
127
Q

Cardiac output would be increased by ________.

A. parasympathetic agonists
B. positive chronotropic agents
C. sympathetic antagonists
D. negative inotropic agents

A

B. positive chronotropic agents

How well did you know this?
1
Not at all
2
3
4
5
Perfectly
128
Q

Cardiovascular changes resulting from moderate exercise include ________.

A. increased total peripheral resistance

B. decreased stroke volume

C. increased cardiac output

D. increased visceral blood flow

A

C. increased cardiac output

How well did you know this?
1
Not at all
2
3
4
5
Perfectly
129
Q

Dizziness during hyperventilation is due to a decrease in cerebral blood flow. The decreased blood flow is caused by ________.

A. decreased respiration
B. vasoconstriction
C. vasodilation
D. decreased stroke volume

A

B. vasoconstriction

How well did you know this?
1
Not at all
2
3
4
5
Perfectly
130
Q

Edema would be induced by all of the following EXCEPT ________.

A. protein malnutrition
B. hypertension
C. lymphatic blockage
D. Hyperthyroidism

A

D. Hyperthyroidism

How well did you know this?
1
Not at all
2
3
4
5
Perfectly
131
Q

Hypertension can be treated by all of the following EXCEPT ________.

A. diuretics
B. ACE inhibitors
C. calcium channel blockers
D. sympathetic agonists

A

D. sympathetic agonists

How well did you know this?
1
Not at all
2
3
4
5
Perfectly
132
Q

Hypertension may cause ________.

A. increased afterload
B. hypertrophy of the ventricles and valve defects
C. broken capillaries in tissues and organs
D. All of the choices are correct.

A

D. All of the choices are correct.

How well did you know this?
1
Not at all
2
3
4
5
Perfectly
133
Q

If the vagus nerve was damaged, you would expect_______.

A. an increase in the responsiveness of the
baroreceptor reflex
B. a decrease in cardiac output
C. an increase in blood pressure
D. a decrease in heart rate

A

A. an increase in the responsiveness of the
baroreceptor reflex

How well did you know this?
1
Not at all
2
3
4
5
Perfectly
133
Q

Norepinephrine is released from ________ neurons, causing heart rate to ________.

A. sympathetic; decrease
B. sympathetic; increase
C. parasympathetic; decrease
D. parasympathetic; increase

A

B. sympathetic; increase

How well did you know this?
1
Not at all
2
3
4
5
Perfectly
134
Q

Stroke volume is directly proportional to ________.

A. total peripheral resistance
B. contractility
C. preload
D. Both preload and contractility are correct

A

D. Both preload and contractility are correct

How well did you know this?
1
Not at all
2
3
4
5
Perfectly
135
Q

The cardiac control center of the ________ coordinates the autonomic innervation of the heart.

A. midbrain
B. pons
C. medulla oblongata
D. cerebrum

A

C. medulla oblongata

How well did you know this?
1
Not at all
2
3
4
5
Perfectly
136
Q

The hormone atrial natriuretic peptide (ANP) functions antagonistically to ________.

A. aldosterone
B. glucocorticoids
C. ADH
D. angiotensin I

A

A. aldosterone

How well did you know this?
1
Not at all
2
3
4
5
Perfectly
137
Q

There is no mixing of blood layers or sounds from vessels during ________ flow.

A. Korotkoff
B. laminar
C. turbulent
D. restricted

A

B. laminar

How well did you know this?
1
Not at all
2
3
4
5
Perfectly
138
Q

Vascular smooth muscle can directly respond to changes in pressure. This type of control is called ________.

A. myogenic
B. vasogenic
C. metabolic
D. extrinsic

A

A. myogenic

How well did you know this?
1
Not at all
2
3
4
5
Perfectly
139
Q

Vasodilation is induced by all of the following EXCEPT ________.

A. increased extracellular K+
B. decreased tissue metabolism
C. increased tissue carbon dioxide
D. decreased tissue oxygen

A

B. decreased tissue metabolism

How well did you know this?
1
Not at all
2
3
4
5
Perfectly
140
Q

What allows cardiac muscle to have sufficient oxygen during systole?

A. Large amount of myoglobin
B. Many mitochondria
C. Many aerobic enzymes
D. All of the choices are correct

A

D. All of the choices are correct

How well did you know this?
1
Not at all
2
3
4
5
Perfectly
141
Q

Which of the following accounts for the increased osmolality with prolonged exercise in a hot environment?

A. The osmoreceptors trigger the thirst mechanism, triggering an increase in blood osmolality.

B. ADH production triggers the increased reabsorption of water in renal tubules trigger the increase in blood osmolality.

C. The increased production of aldosterone leads to an increase in the blood osmolality.

D. The decreased blood volume due to increased sweating and lack of water intake increase the blood osmolality

A

D. The decreased blood volume due to increased sweating and lack of water intake increase the blood osmolality

How well did you know this?
1
Not at all
2
3
4
5
Perfectly
142
Q

The ejection fraction for an individual whose stroke volume is 45 ml of blood would be ________. This would ________ meet the body’s demand for blood flow.

A. 10%; inadequately
B. 75%; adequately
C. 40%; inadequately
D. 60%; adequately

A

C. 40%; inadequately

How well did you know this?
1
Not at all
2
3
4
5
Perfectly
143
Q

What type of receptors do norepinephrine and epinephrine bind in the heart?

A. Muscarinic
B. α-adrenergic
C. Nicotinic
D. β1-adrenergic

A

D. β1-adrenergic

How well did you know this?
1
Not at all
2
3
4
5
Perfectly
144
Q

What is the average resting heart rate for an adult?

A. 70 beats/min
B. 60 beats/min
C. 75 beats/min
D. 80 beats/min

A

A. 70 beats/min

How well did you know this?
1
Not at all
2
3
4
5
Perfectly
145
Q

The opposing forces that affect fluid movements across a capillary are known as ________.

A. oncotic pressures
B. starling forces
C. colloid osmotic pressures
D. None of the choices are correct.

A

B. starling forces

How well did you know this?
1
Not at all
2
3
4
5
Perfectly
146
Q

Which of the following is NOT a mechanism by which angiotensin II raises blood pressure?

A. Causing vasodilation in arterioles
B. Stimulating the thirst center in the hypothalamus
C. Increasing peripheral resistance
D. Stimulating production of aldosterone

A

A. Causing vasodilation in arterioles

How well did you know this?
1
Not at all
2
3
4
5
Perfectly
147
Q

Angiotensin I is formed from the plasma protein ________ by the action of the enzyme ________.

A. angiotensinogen; renin
B. angiotensin II; renin
C. angiotensinogen; ACE
D. renin; AC

A

A. angiotensinogen; renin

How well did you know this?
1
Not at all
2
3
4
5
Perfectly
148
Q

Nitric oxide (NO) causes ________.
A. vasoconstriction
B. vasodilation
C. no effect

A

B. vasodilation

How well did you know this?
1
Not at all
2
3
4
5
Perfectly
149
Q

During exercise, all of the following occur EXCEPT ________.

A. increased oncotic pressure
B. visceral vasoconstriction
C. increased systolic blood pressure
D. increased cardiac output

A

A. increased oncotic pressure

How well did you know this?
1
Not at all
2
3
4
5
Perfectly
150
Q

During maximal exercise, the skeletal muscles can receive as much as ________ of the total blood flow.

A. 85%
B. 65%
C. 45%
D. 35%

A

A. 85%

How well did you know this?
1
Not at all
2
3
4
5
Perfectly
151
Q

The sounds heard during the first phase of blood-pressure measurement are ________.

A. snapping sounds
B. thumping sounds
C. muffled thumping sounds
D. murmurs

A

A. snapping sounds

How well did you know this?
1
Not at all
2
3
4
5
Perfectly
152
Q

Left side heart failure may be caused by ________.
A. myocardial infarction
B. aortic valve stenosis
C. incompetent aortic and mitral valves
D. All of the choices are correct.

A

D. All of the choices are correct.

How well did you know this?
1
Not at all
2
3
4
5
Perfectly
153
Q

The ejection fraction is ________.

A. increased by parasympathetic agonists

B. the ratio of stroke volume to end-diastolic volume

C. the ratio of end-diastolic volume to end-systolic volume

D. decreased by sympathetic agonists

A

B. the ratio of stroke volume to end-diastolic volume

How well did you know this?
1
Not at all
2
3
4
5
Perfectly
154
Q

Imagine a patient with a stiff, non-compliant left ventricle is prescribed a medication that acts as an agonist to norepinephrine. The medication would be effective because ________.

A. blood pressure would increase, moving blood through the circulatory system more rapidly

B. cardiac output would increase, forcing the ventricle to become more compliant

C. heart rate would increase, compensating for the reduced stroke volume

D. heart rate would decrease, reducing the workload on the heart

A

C. heart rate would increase, compensating for the reduced stroke volume

155
Q

The ________ is equal to the hydrostatic pressure of the blood in the capillaries minus the hydrostatic pressure of tissue interstitial fluid outside the capillaries.

A. net filtration pressure

B. blood pressure

C. osmotic pressure

D.None of the choices are correct.

A

A. net filtration pressure

156
Q

Angiotensin-converting enzyme (ACE) inhibitors like captopril block the formation of angiotensin II, thus reducing ________.

A. renal excretion
B. renal reabsorption
C. vasoconstriction
D. vasodilation

A

C. vasoconstriction

157
Q

Essential hypertension may result from ________.

A. a diet high in sodium
B. decreased sympathetic vasoconstriction
C. decreased ADH secretion
D. decreased release of endothelin

A

A. a diet high in sodium

158
Q

The type of shock resulting from lipopolysaccharides is termed ________ shock.

A. Hypovolemic
B. Neurogenic
C. Cardiogenic
D. Septic

A

D. Septic

159
Q

Increased baroreceptor response will lead to which of the following homeostatic responses?

A. Increased epinephrine release

B. Decreased total peripheral resistance

C. Decreased acetylcholine release

D. Increased ADH and aldosterone release

E. Increased cardiac rate

A

B. Decreased total peripheral resistance

160
Q

Which blood vessels have the highest cross-sectional area?

A. Capillaries

B. Venules

C. Veins

D. Arterioles

A

A. Capillaries

161
Q

Sweat glands can also secrete ________, which stimulates vasodilation of skin arterioles to help reduce body temperature.

A. nitric oxide
B. adenosine
C. bradykinin
D. prostaglandin E2

A

C. bradykinin

162
Q

Blood flow would be increased by __________.

A. angiotensin II
B. Thromboxane A2
C. Antihistamines
D. Histamines

A

D. Histamines

163
Q

According to the Frank-Starling Law of the heart, as contraction strength increases the stroke volume ________.

A. Increases
B. Decreases
C. Shows no change

A

A. Increases

164
Q

In response to increased ventricular filling, ________.

A. myocardial contractility in decreases
B. sympathetic nerves release norepinephrine
C. total peripheral resistance increases
D. myocardial cells stretch

A

D. myocardial cells stretch

165
Q

The ability to distend with pressure increases is called ________.

A. Inotropism
B. Chronotropism
C. Compliance
D. Oncotism

A

C. Compliance

166
Q

ADH would be considered a(n) ________ regulator of blood flow.

A. Extrinsic
B. Both extrinsic and hormonal are correct
C. Hormonal
D. Intrinsicliver

A

B. Both extrinsic and hormonal are correct

167
Q

Dangerously low blood pressure that results from infection is ________ shock.

A. Cardiogenic
B. Neurogenic
C. Anaphylactic
D. Septic

A

D. Septic

168
Q

Norepinephrine and epinephrine open ________ channels inducing a faster diastolic depolarization.

A.HCN
B.fast Ca2+
C.fast Na+
D.voltage-gated Ca2+

A

A.HCN

169
Q

Net filtration would be decreased by ________.

A.liver damage
B.increased cellular metabolism
C.hypotension
D.increased hepatic protein synthesis

A

C.hypotension

170
Q

Elephantiasis is caused by ________.

A.myxedema
B.obstruction of lymphatic drainage
C.decreased plasma protein concentration
D.high arterial blood pressure

A

B.obstruction of lymphatic drainage

171
Q

A decrease in total peripheral resistance would cause blood flow to ________.
A.decrease
B.remain unchanged
C.increase

A

C.increase

172
Q

Control of blood flow to skeletal muscles during exercise is mainly under ________ control.

A.intrinsic myogenic
B.hormonal
C.intrinsic metabolic
D.extrinsic metabolic

A

C.intrinsic metabolic

173
Q

Stroke volume is inversely proportional to ________.

A. preload
B. contractility
C. Both preload and contractility are correct.
D. total peripheral resistance

A

D. total peripheral resistance

174
Q

Blood volume would be increased by ________.

A. increased renin secretion
B. increased atrial natriuretic factor secretion
C. decreased vasopressin secretion
D. decreased aldosterone secretion

A

A. increased renin secretion

175
Q

What is the main force that causes fluids to enter the venous end of a capillary?

A. Hydrostatic pressure in the capillary
B. Colloid osmotic pressure in the capillary (blood plasma)
C. Hydrostatic force in the interstitial fluid
D. Colloid osmotic pressure in the interstitial fluid

A

B. Colloid osmotic pressure in the capillary (blood plasma)

176
Q

Increased blood flow resistance is caused by ________.

A. increased blood pressure
B. increased blood vessel length
C. vasodilation
D. decreased blood viscosity

A

B. increased blood vessel length

177
Q

If atrial stretch receptors are stimulated, you would NOT expect ________ as a result.

A. increased secretion of ANP
B. decreased vascular volume
C. decreased water excretion
D. inhibition of ADH

A

C. decreased water excretion

178
Q

Expanded blood volume stimulates stretch receptors in the atria of the heart, causing increased secretion of ________.

A. parasympathetic antagonists
B. aldosterone
C. atrial natriuretic peptide
D. epinephrine

A

D. epinephrine

179
Q

During congestive heart failure______.

A. cardiac output is increased
B. high blood Ca2+ may cause cardiac arrest in systole
C. high blood K+ may cause cardiac arrest in systole
D. ejection fraction is increased

A

B. high blood Ca2+ may cause cardiac arrest in systole

180
Q

Stroke volume is a direct result of all of the following EXCEPT ________.

A. Cardiac contractility
B. End-diastolic volume
C. Total peripheral resistance
D. Emotional stress

A

D. Emotional stress

181
Q

Endothelin causes ________.

A. No effect
B. Vasoconstriction
C. vasodilation

A

B. Vasoconstriction

182
Q

Which type of muscle has the greatest capillary density

A. Skeletal
B. Cardiac
C. Smooth

A

B. Cardiac

183
Q

Cardiogenic shock can result from ______.

A. Parasympathetic antagonists

B. Myocardial infarction

C. Increased cardiac output

D. Increased sympathetic outflow to the heart

A

B. Myocardial infarction

184
Q

Which is NOT a symptom of preeclampsia

A. Hypertension
B. Edema
C. Proteinuria
D. Increased vasodilation

A

D. Increased vasodilation

185
Q

Blood flow is increased by _____.

A. Vasoconstriction
B. Vasodilation
C. Edema
D. Hypotension

A

B. Vasodilation

186
Q

The volume of blood in the ventricles at the end of diastole is known as the ________.

A. end-diastolic ADH secretionvolume (EDV)
B. after-load
C. stroke volume (SV)
D. pre-diastolic volume (PDV)

A

A. end-diastolic ADH secretionvolume (EDV)

187
Q

The steroid hormone secreted by the adrenal cortex, which stimulates salt reabsorption in the kidneys is ________.

A. angiotensin
B. ADH
C. renin
D. Aldosterone

A

D. Aldosterone

188
Q

Acetylcholine binds to ________ receptors of the heart and causes the opening of ________ channels.

A. nicotinic; K+
B. muscarinic; Na+
C. nicotinic; Na+
D. muscarinic; K+

A

D. muscarinic; K+

189
Q

An increase in blood volume in the left atrium will ________ ADH secretion.

A. have no effect on
B. inhibit
C. Stimulate

A

B. inhibit

190
Q

The last Korotkoff sound occurs when the blood pressure is equal to ________ pressure.
A. atmospheric
B. pulmonary
C. diastolic
D. Systolic

A

C. diastolic

191
Q

Why is pulse pressure a better indicator of cardiovascular health than blood pressure alone?

A. It takes into account the fact that diastole is longer than systole.

B. It accounts for the perfusion of blood to the visceral organs.

C. It shows the summation of both systolic and diastolic pressures to indicate overall blood volume.

D. It is a better predictor for essential hypertension in patients.

A

B. It accounts for the perfusion of blood to the visceral organs.

192
Q

Sympathetic stimulation of the heart will ___________.

A. Increase the rate of depolarization in the ventricles

B. Increase conduction rate between the SA and AV nodes

C. Increase the strength of myocardial contraction

D. All of the choices are correct

A

D. All of the choices are correct

193
Q

Muscle length has a more pronounced effect on contraction strength in cardiac muscle than in skeletal muscle. This is because cardiac muscle has a greater sensitivity to ________.

A. Parasympathetic antagonists
B. Depolarization
C. Calcium
D. Lactic acid

A

C. Calcium

194
Q

How much of the total body water is generally found in the blood plasma?

A. 33%
B. 6.7%
C. 67%
D. 26.3%

A

B. 6.7%

195
Q

The events listed below are all components of respiration EXCEPT ____.

A. Ventilation
B. Gas exchange
C. Oxygen utilization
D. Speech

A

D. Speech

196
Q

What phospholipid decreases the surface tension of the alveoli?

A. Surfactant
B. Saliva
C. Lymph
D. mucus

A

A. Surfactant

197
Q

Ventilation would be decreased by decreasing the activity of ____.

A. Type II alveolar cells
B. Type I alveolar cells
C. Alveolar macrophages
D. None of the choices are correct

A

A. Type II alveolar cells

198
Q

The tendency of lungs to return to their initial size after stretching is ____.

A. Compliance
B. Elasticity
C. Surface tension
D. None of the choices are correct

A

B. Elasticity

199
Q

Forced (deep) inspiration occurs with the contraction of the ____.

A. Parasternal intercostals
B. Scalenes
C. Internal intercostals
D. Rectus abdominis

A

B. Scalenes

200
Q

Which of the following may result from emphysema?

A. Reduced gas exchange surface area
B. Air trapping
C. Cor pulmonale
D. All of the choices are correct

A

D. All of the choices are correct

201
Q

Why would an individual with atopic allergic asthma experience difficulty breathing?

A. Mast cells, which stimulate vasodilation of the airways, are less active in persons with allergies

B. IgE antibodies bind to the inhaled antigens, causing an inflammatory reaction that decreases gas exchange

C. Leukotrienes cause constriction of the bronchioles, producing airway obstruction

A

C. Leukotrienes cause constriction of the bronchioles, producing airway obstruction

202
Q

You open a can of carbonated soda and pour it into a glass. The carbon dioxide formed bubbles because the partial pressure of carbon dioxide in the air above the soda ____, a process explained by ____.

A. Decreased; Dalton’s
B. Decreased; Henry’s
C. Increased; Boyle’s
D. Increased; Charles’

A

B. Decreased; Henry’s

203
Q

Normal alveolar PO2 is ____.

A. 105 mm Hg
B. 46 mm Hg
C. 100 mm Hg
D. 40 mm Hg

A

A. 105 mm Hg

204
Q

Normal arterial PO2 is ____.

A. 40 mm Hg
B. 85 mm Hg
C. 100 mm Hg
D. 46 mm Hg

A

C. 100 mm Hg

205
Q

Which law states that the total pressure of a gas mixture is equal to the sum of the pressures that each gas in the mixture would exert independently?

A. Boyle’s Law
B. Henry’s Law
C. Dalton’s Law
D. Charles’ Law

A

C. Dalton’s Law

206
Q

Pulmonary arterioles ____ and system arterioles ____ when PO2 is low.

A. Dilate; dilate
B. Dilate; constrict
C. Constrict; dilate
D. Constrict; constrict

A

C. Constrict; dilate

207
Q

The primary drive to breathe is elicited by which of the following?

A. Increased PO2
B. Increased PCO2
C. Reduced PO2
D. Reduced PCO2

A

B. Increased PCO2

208
Q

The rhythmicity center is located in the ____.

A. Pons
B. Medulla oblongata
C. Cerebral cortex

A

B. Medulla oblongata

209
Q

At rest, normal oxygen unloading is ____.

A. 75%
B. 22%
C. 39%
D. 97%

A

B. 22%

210
Q

Assuming you have normal respiratory function at sea level, start holding your breath. Which of the following could be true in 60 seconds?

A. Alveolar PO2 = 105 mmHg

B. Venous PO2 = 50 mmHg

C. Arterial PCO2 = 50 mmHg

D. The oxygen-hemoglobin dissociation curve is shifting to the left

A

C. Arterial PCO2 = 50 mmHg

211
Q

2,3-DPG comes from ____.

A. Type II alveolar cells
B. Aerobic respiration in red blood cells
C. Tissues with high amounts of oxygen
D. Anaerobic respiration in red blood cells

A

D. Anaerobic respiration in red blood cells

212
Q

The majority of carbon dioxide is transported in the blood as ____.

A. Bicarbonate ion
B. Carboxyhemoglobin
C. Dissolved carbon dioxide in the blood
D. carbaminohemoglobin

A

A. Bicarbonate ion

213
Q

Which of the following changes would occur during exercise?

A. Decreased oxygen delivery to muscles
B. Decreased ventilation
C. Increased oxygen extraction by muscles
D. Increased blood gasses

A

C. Increased oxygen extraction by muscles

214
Q

Acclimatization to altitude ____.

A. Stimulates increased blood cell synthesis
B. Decreases ventilation
C. Increases the affinity of hemoglobin for oxygen
D. All of the choices are correct

A

A. Stimulates increased blood cell synthesis

215
Q

Fluid secretion by lung cells is due to ________ alveolar cells.
A. active transport of Cl- out of
B. passive transport of HCO3- out of
C. active transport of Na+ out of
D. active transport of Na+ into

A

A. active transport of Cl- out of

216
Q

Respiratory Distress Syndrome (RDS) is a condition associated with premature babies who lack ________.

A. histamine
B. surfactant
C. a-antitrypsin
D. type I alveolar cells

A

B. surfactant

217
Q

Which of the following is an inflammatory cell associated with COPD but NOT asthma?

A. Eosinophils
B. Mast cells
C. Helper T cells
D. Cytotoxic T cells

A

D. Cytotoxic T cells

218
Q

Pulmonary fibrosis may be caused by ________.
A. increased mucus production
B. breathing in coal dust
C. smoking
D. allergic reactions

A

B. breathing in coal dust

219
Q

Which law states that the amount of gas dissolved in a liquid is directly proportional to the partial pressure of the gas?
A. Charles’ law
B. Henry’s law
C. Boyle’s law
D. Laplace’s law

A

B. Henry’s law

220
Q

Blood flow is greatest at the ________ of the lungs and perfusion is greatest at the ________ of the lungs.
A. apex; base
B. base; apex
C. apex; apex
D. base; base

A

D. base; base

221
Q

Hyperbaric oxygen therapy ________.
A. exposes patients to high oxygen under low pressure
B. can be used to promote wound healing
C. is used to treat nitrogen narcosis
D. would increase the time required to recover from decompression sickness

A

B. can be used to promote wound healing

222
Q

What type of receptors in the wall of the larynx and lungs, cause a person to cough in response to components of smoke and smog?

A. Pulmonary stretch receptors
B. Medullary chemoreceptors
C. Aortic bodies
D. Irritant receptors

A

D. Irritant receptors

223
Q

The dorsal respiratory group in the medulla oblongata is involved with ________.
A. Both inspiration and expiration are correct.
B. breathing rhythm
C. inspiration
D. expiration

A

C. inspiration

224
Q

The form of hemoglobin with iron in a reduced state and bonded to carbon monoxide is ________.

A. methemoglobin
B. deoxyhemoglobin
C. carboxyhemoglobin
D. oxyhemoglobin

A

C. carboxyhemoglobin

225
Q

The highest oxygen affinity is demonstrated by ________.

A. myoglobin
B. hemoglobin A
C. hemoglobin F
D. hemoglobin S

A

A. myoglobin

226
Q

The affinity of hemoglobin for oxygen ________.

A. decreases as the height above sea level increases
B. is greater than the affinity for carbon monoxide
C. is increased in methemoglobin
D. is increased in response to metabolic alkalosis

A

D. is increased in response to metabolic alkalosis

227
Q

During muscular contractions associated with inhalation, what would happen if intrapleural pressure were to be less than intrapulmonary pressure?

A. Thoracic volume increase with lung collapse

B. Thoracic volume decrease with lung collapse

C. Lung volume would remain unchanged

D. Thoracic volume increase with lung inflation

A

D. Thoracic volume increase with lung inflation

228
Q

Intrapleural pressure ________ during expiration

A. Remains unchanged
B. Increases
C. Decreases

A

B. Increases

229
Q

Measure of the distensibility of the lungs is ________

A. Compliance
B. Elasticity
C. Surface tension
D. None of the choice are correct

A

A. Compliance

230
Q

Contraction of the ___________ will stimulate inspiration

A. Diaphragm
B. External intercostals
C. Parasternal intercostals
D. All of the choices are correct

A

D. All of the choices are correct

231
Q

Cessation of breathing is known as _____

A. Pneumothorax
B. Apnea
C. Dyspnea
D. Eupnea

A

B. Apnea

232
Q

Due to acute trauma and resulting damage of the diaphragm, the ____ will need to become more active in order to aid someone during ______

A. External intercostals; expiration

B. Rectus abdominis; inspiration

C. Scalenes and pectoralis minor; inspiration

D. Sternocleidomastoid and parasternal muscle; expiration

A

C. Scalenes and pectoralis minor; inspiration

233
Q

Breathing 100% oxygen will NOT _____.

A. Significantly increase the total oxygen content of whole blood

B. Increase the amount of oxygen in red blood cells

C. Significantly increase the oxygen delivery to tissues

D. Both increase the amount of oxygen in red blood cells and significantly increase the total oxygen content of whole blood

A

D. Both increase the amount of oxygen in red blood cells and significantly increase the total oxygen content of whole blood

234
Q

Normal alveolar PCO2 is _____.

A. 46 mm Hg
B. 100 mm Hg
C. 105 mm Hg
D. 40 mm Hg

A

D. 40 mm Hg

235
Q

Hyperventilation is stimulated by _____

A. Hypercapnia
B. Increased activity of the apneustic center
C. Decreased contraction of the scalenes
D. Hypoxemia

A

A. Hypercapnia

236
Q

The form of hemoglobin with iron in a reduced state and not bonded to oxygen is ____

A. Methemoglobin
B. Carboxyhemoglobin
C. Oxyhemoglobin
D. Deoxyhemoglobin

A

D. Deoxyhemoglobin

237
Q

The form of hemoglobin with iron in a reduced state and bonded to oxygen is ____.

A. Methemoglobin
B. Carboxyhemoglobin
C. Oxyhemoglobin
D. Deoxyhemoglobin

A

C. Oxyhemoglobin

238
Q

The metabolic regulation of blood pH occurs in _____

A. The lungs
B. The liver
C. All organs
D. The kidneys

A

D. The kidneys

239
Q

Many athletes train at higher altitudes immediately prior to an athletic competition. Select the scenario that correctly identifies how acclimatization to a higher elevation would improve endurance performance.

A. A hypoxic ventilatory response occurs that remains in place for a few weeks after returning to a lower altitude

B. Erythropoietin release will increase; consequently, the athlete will have a greater aerobic capacity when returning to the lower altitude

C. Increased ventilation at the higher altitudes will improve tidal volume when returning to the lower altitude

D. Hemoglobin’s affinity for oxygen will increase; therefore, tissues will receive more oxygen during exercise

A

B. Erythropoietin release will increase; consequently, the athlete will have a greater aerobic capacity when returning to the lower altitude

240
Q

A pulmonologist is examining a patient who has been smoking cigarettes for several years. The patient’s history includes frequent upper and lower respiratory tract infections. What explains the patient’s history?

A. Gas exchange is diminished by cigarette smoking, which increases the susceptibility to infection.

B. Smoking increases ciliary movement of mucus in the airways, causing pulmonary congestion.

C. Smoking suppresses the innate defenses in the conducting zone of the respiratory system.

D. Macrophage activity is increased by cigarette smoking, causing inflammation.

A

C. Smoking suppresses the innate defenses in the conducting zone of the respiratory system.

241
Q

The volume of gas remaining in the lungs after a maximum expiration is the ________.

A. inspiratory reserve volume
B. residual volume
C. vital capacity
D. tidal volume

A

B. residual volume

242
Q

Pulmonary circulation is a ________ resistance and ________ pressure pathway.

A. high; low
B. low; high
C. low; low
D. high; high

A

C. low; low

243
Q

When alveolar ventilation increases, the perfusion of pulmonary arterioles will ________.

A. remain unchanged
B. decrease
C. Increase

A

C. Increase

244
Q

Central chemoreceptors in the medulla oblongata directly detect changes in the pH of the ________.

A. lymph
B. cerebrospinal fluid
C. blood
D. air

A

B. cerebrospinal fluid

245
Q

A decrease in temperature ________.

A. will shift the oxygen dissociation curve to the left
B. will not shift the oxygen dissociation curve
C. will shift the oxygen dissociation curve to the right

A

A. will shift the oxygen dissociation curve to the left

246
Q

The exchange of chloride ions for bicarbonate through tissue capillaries is called the ________.

A. Bohr effect
B. oxygen toxicity
C. chloride shift
D. acidosis

A

C. chloride shift

247
Q

The conducting zone contains all of the following EXCEPT the ________.

A. ​​respiratory bronchioles
B. terminal bronchioles
C. larynx
D. primary bronchi

A

A. ​​respiratory bronchioles

248
Q

Acute respiratory distress syndrome causes a protein-rich fluid to accumulate rapidly in the lungs. Predict the consequences of this syndrome if medical intervention does not occur.

A. Tissues throughout the body would experience ischemia due to decreased oxygen delivery.

B. All of the choices are correct.

C. Air would accumulate in the intrapleural space, interfering with gas exchange.

D. Air would be unable to move through the conducting zone.

A

A. Tissues throughout the body would experience ischemia due to decreased oxygen delivery.

249
Q

How does surfactant reduce the surface tension of water in the lungs?

A. Decomposes water
B. Reduces hydrogen bonding between water molecules
C. Covers the alveoli
D. All of the choices are correct.

A

B. Reduces hydrogen bonding between water molecules

250
Q

The ________ separates the abdominal and thoracic cavities.

A.lungs
B.liver
C.diaphragm
D.rib cage

A

C.diaphragm

251
Q

The amount of a given gas dissolved in the blood ________.

A. is directly proportional to the partial pressure of the gas

B. increases at higher altitudes

C. is described primarily by Boyle’s law
​​
D. All of the choices are correct.

A

A. is directly proportional to the partial pressure of the gas

252
Q

Identify the true statement regarding gas concentrations in the pulmonary system.

A.Blood in the pulmonary veins is low in oxygen.

B.Blood in the pulmonary veins is high in carbon dioxide.

C.The oxygen concentration of inspired air is higher than that of alveolar air.

D.All of the choices are correct.

A

C.The oxygen concentration of inspired air is higher than that of alveolar air.

253
Q

Quiet expiration is caused by ________.

A.contraction of the internal intercostals

B.contraction of the scalenes

C.contraction of the external intercostals

D.lung recoil and increased intrapulmonary pressure

A

D.lung recoil and increased intrapulmonary pressure

254
Q

The foramen ovale ________.

A.normally shunts blood between the right and left ventricles

B.has no importance in fetal respiratory physiology

C.closes after birth due to decreased pulmonary vascular resistance

D.normally shunts blood between the pulmonary artery and aorta

A

C.closes after birth due to decreased pulmonary vascular resistance

255
Q

Normal venous PCO2 is ________.

A. 46 mm Hg
B. 40 mm Hg
C. 100 mm Hg
D. 105 mm Hg

A

A. 46 mm Hg

256
Q

The normal partial pressure differences between inspired air and alveolar gas for O2, CO2, H2O, and N2 can be explained because of ________.

A. net movements of CO2 toward the alveoli, O2 toward the alveoli, and H2O is added along the respiratory tract

B.net movement of CO2 and N2 toward the alveoli and H2O movements toward the alveoli as respiratory water

C.net movements of CO2, O2, and H2O into the alveoli from blood capillaries

D.net movements of CO2 toward the alveoli with net movements of O2 and H2O into the capillaries

A

A. net movements of CO2 toward the alveoli, O2 toward the alveoli, and H2O is added along the respiratory tract

257
Q

What is the normal value of arterial percent hemoglobin saturation?

A. 100%
B. 90%
C. 86%
D. 97%

A

D. 97%

258
Q

Which of the following will decrease the affinity of hemoglobin for oxygen?

A. Increased pH, increased temperature, or decreased 2,3-DPG

B. Increased pH, decreased temperature, or increased 2,3-DPG

C. Decreased pH, increased temperature, or increased 2,3-DPG

D. Decreased pH, decreased temperature, or decreased 2,3-DPG

A

C. Decreased pH, increased temperature, or increased 2,3-DPG

259
Q

The affinity of hemoglobin for oxygen is ________ as the partial pressure of oxygen is raised.

A. Unchanged
B. Decreased
C. Increased

A

C. Increased

260
Q

In anemia, 2,3-DPG is ________ and oxygen affinity is ________.

A. increased; decreased
B. decreased; decreased
C. increased; increased
D. decreased; increased

A

A. increased; decreased

261
Q

The form of hemoglobin with iron in an oxidized state is ________.

A. carboxyhemoglobin
B. methemoglobin
C. deoxyhemoglobin
D. Oxyhemoglobin

A

B. methemoglobin

262
Q

The enzyme ________ catalyzes the formation of H2CO3 from CO2 and water.

A. carbonic anhydrase
B. renin
C. lactate dehydrogenase
D. nitric oxide synthase

A

A. carbonic anhydrase

263
Q

Metabolic alkalosis ________.

A. is induced by hypoventilation

B. may be caused by excessive vomiting

C. occurs when arterial pH is less than 7.4

D.occurs when the partial pressure of oxygen is decreased

A

B. may be caused by excessive vomiting

264
Q

What law states that the pressure of a given quantity of gas is inversely proportional to its volume?

A. Henry’s Law

B. Boyle’s Law

C. Charles’ law

D. Dalton’s Law

A

B. Boyle’s Law

265
Q

Rebecca’s inspiratory reserve volume = 5, vital capacity= 8, residual volume = 1, and expiratory reserve volume = 2. Her tidal volume would be equal to ________ and her total lung capacity would be ________.

A. Unable to determine with the given data.
B. 3; 7
C. 1; 9
D. 2; 10

A

C. 1; 9

266
Q

What condition is caused by large amounts of nitrogen dissolving into the blood due to hyperbaric conditions?

A. Emphysema
B. Oxygen toxicity
C. Nitrogen narcosis
D. Decompression sickness

A

C. Nitrogen narcosis

267
Q

Hemoglobin F ________.

A. is increased in sickle-cell anemia

B. contains two alpha and two gamma chains

C. binds large amounts of 2,3-DPG

D. has a lower affinity for oxygen than hemoglobin A

A

B. contains two alpha and two gamma chains

268
Q

Carbon dioxide ________ oxygen unloading and oxygen unloading ________ carbon dioxide transport.

A. increases; worsens
B. increases; improves
C. decreases; worsens
D. decreases; improves

A

B. increases; improves

269
Q

A pH above 7.45 is considered ________.

A. acidosis
B. metabolic
C. alkalosis
D. Normal

A

C. alkalosis

270
Q

Peripheral chemoreceptors that can detect changes in blood pH are located in the ________.

A. Medulla oblongata
B. Aortic and carotid bodies
C. Lungs
D. pons

A

B. Aortic and carotid bodies

271
Q

Carbon monoxide is lethal because it ________.

A. increases oxygen unloading at the cells

B. binds hemoglobin, preventing oxygen-binding

C. reduces CO2 and slows breathing

D. increases CO2 and causes seizures

A

B. binds hemoglobin, preventing oxygen-binding

272
Q

Hypoventilation can correct _________.

A. Metabolic alkalosis
B. Metabolic acidosis
C. Respiratory alkalosis
D. Respiratory acidosis

A

A. Metabolic alkalosis

273
Q

During inspiration, ________.

A. transpulmonary pressure increases

B. alveolar pressure exceeds atmospheric pressure

C. intrapulmonary pressure is less than atmospheric pressure

D. the diaphragm relaxes

A

C. intrapulmonary pressure is less than atmospheric pressure

274
Q

Normal venous PO2 is ________.

A. 46 mm Hg
B. 100 mm Hg
C. 85 mm Hg
D. 40 mm Hg

A

D. 40 mm Hg

275
Q

Decompression sickness is caused by ascending to sea level too quickly which results in bubbles of ________ to form in the blood.

A. Oxygen
B. Nitrogen
C. Carbon dioxide
D. Carbon monoxide

A

B. Nitrogen

276
Q

The ventilation/perfusion ratio is ________ at the apex of the lungs.

A. The same as the base
B. Highest
C. Lowest

A

B. Highest

277
Q
  1. What is the condition of having low blood oxygen levels?

A. Hypercapnia
B. Hypoxemia
C. Hypoxia
D. Hypocapnia

A

B. Hypoxemia

278
Q

If PO2 and hemoglobin content of blood is normal, how much oxygen is the blood carrying?

A. 0.3 mL O2/ 100 mL blood
B. 10 mL O2/ 100 mL blood
C. 20 mL O2/ 100 mL blood
D. 35 mL O2/ 100 mL blood

A

C. 20 mL O2/ 100 mL blood

279
Q

According to the Bohr effect, as pH is lowered, the affinity of hemoglobin for oxygen ________.

A. Remains unchanged
B. Increases
C. Decreases

A

C. Decreases

280
Q

In assessing a mechanical prototype to demonstrate lung function, it is suggested that the final version of synthetic lung material will need to be easier to inflate and easier to deflate in order to be cost-effective. Which will need to be true?

A. More compliant and more elastic

B. Less compliant and less elastic

C. Less compliant and more elastic

D. More compliant and less elastic

A

A. More compliant and more elastic

281
Q

Quiet inspiration will ________ thoracic and lung volume and ________ intrapulmonary pressure.

A. increase; increase
B. increase; decrease
C. decrease; decrease
D. decrease; increase

A

B. increase; decrease

282
Q

Limits on stretching the lungs are due to the ________.

A. Haldane effect
B. law of Laplace
C. Hering-Breuer reflex
D. None of the choices are correct

A

C. Hering-Breuer reflex

283
Q

The Bohr effect describes the ________.

A. effect of pH on the affinity of hemoglobin for carbon dioxide

B. effect of solubility on the amount of gas dissolved in blood

C. mechanism of ventilation

D. effect of pH on the affinity of hemoglobin for oxygen

A

D. effect of pH on the affinity of hemoglobin for oxygen

284
Q

Diffusion rate across the respiratory membrane is rapid because ________.

A. there are about 750 square feet of alveoli membrane
B. alveoli are one cell thick
C. the air-blood barrier is two cells thick
D. All of the choices are correct.

A

D. All of the choices are correct.

285
Q

Breathing 100% oxygen will ________.

A. significantly increase the oxygen delivery to tissues

B. not change the amount of oxygen dissolved in the plasma

C. significantly increase the total oxygen content of whole blood

D. increase the amount of oxygen in red blood cells

A

A. significantly increase the oxygen delivery to tissues

286
Q

An increase in 2,3-diphosphoglyceric acid (2,3-DPG) will ________ the affinity of hemoglobin for oxygen.

A. decrease
B. have no effect on
C. increase

A

A. decrease

287
Q

Which of the following conditions is characterized by high red blood cell counts?

A. Polycythemia
B. Uremia
C. Anemia
D. Leukemia

A

A. Polycythemia

288
Q

Where does the reverse chloride shift occur?

A. Pulmonary capillaries
B. Venules
C. Tissue capillaries
D. Arterioles

A

A. Pulmonary capillaries

289
Q

Gas X makes up 15% of a gas mix, at 760 mm Hg, the partial pressure of gas X would be ________.

A. 646 mm Hg
B. 11400 mm Hg
C. 114 mm Hg
D. 1140 mm Hg

A

C. 114 mm Hg

290
Q

A family of hemoglobin diseases found primarily in people of Mediterranean ancestry is ________.

A. leukemia
B. sickle-cell anemia
C. myoglobinemia
D. thalassemia

A

D. thalassemia

291
Q

The total amount of gas in the lungs after a maximum inspiration is the ________.

A. Vital capacity
B. Total lung capacity
C. Tidal volume
D. Functional residual capacity

A

B. Total lung capacity

292
Q

The actions of the kidney include regulation of ________.
A. blood plasma pH
B. blood plasma volume
C. blood plasma electrolyte concentration
D. All of the choices are correct.

A

D. All of the choices are correct.

293
Q

The process of urination is called ________.
A. incontinence
B. deglutition
C. glomerular filtration
D. micturition

A

D. micturition

294
Q

Incontinence could develop as a result of ________.

A. pressure on the detrusor muscle
B. poor bladder compliance
C. damage to the external urethral sphincter
D. All of the choices are correct.

A

D. All of the choices are correct.

295
Q

Where is the micturition center?
A. Medulla oblongata
B. Urinary bladder
C. Pons
D. S2–S4 of the spinal cord

A

C. Pons

296
Q

What is the net filtration pressure of the glomerular capillaries?
A. 10 mm Hg inward
B. 10 mm Hg outward
C. 15 mm Hg inward
D. 15 mm Hg outward

A

B. 10 mm Hg outward

297
Q

The ability of the kidneys to maintain a relatively constant GFR despite fluctuating blood pressures is called renal ________.
A. plasma threshold
B. transport maximum
C. plasma clearance
D. autoregulation

A

D. autoregulation

298
Q

Which of the following is NOT a potential filtration barrier in the glomerular capsule?
A. Glomerular basement membrane
B. Capillary fenestrae
C. Slit diaphragm
D. Parietal layer of the capsule

A

D. Parietal layer of the capsule

299
Q

The tubular filtrate osmolality ________ as it flows through the ascending limb of the nephron loop.
A. increases
B. remains unchanged
C. decreases

A

C. decreases

300
Q

The ________ limb of the nephron loop is impermeable to water.
A. ascending
B. descending
C. both limbs
D. neither limb

A

A. ascending

301
Q

Na+ moves by ________ from the filtrate into the epithelial cells of the proximal convoluted tubule.
A. diffusion
B. pinocytosis
C. Na+/K+ pump
D. active transport

A

A. diffusion

302
Q

The ________ surface of the simple cuboidal epithelia in the proximal convoluted tubule contains microvilli.
A. basal
B. lateral
C. apical
D. basolateral

A

C. apical

303
Q

Countercurrent multiplication occurs by the actions of the ________.
A. nephron loop
B. vasa recta
C. Both vasa recta and nephron loop are correct.
D. peritubular capillaries

A

C. Both vasa recta and nephron loop are correct.

304
Q

If blood glucose was elevated in the urine, which of the following changes to occur in the body?
A. Increased glucose carriers in the renal tubules

B. Increased aldosterone production from the adrenal cortex

C. Increased GFR

D. Increased permeability to water in the collecting duct

A

D. Increased permeability to water in the collecting duct

305
Q

The movement of molecules and ions from the peritubular capillaries into interstitial fluid and then into the nephron tubule is called ________.
A. filtration
B. excretion
C. reabsorption
D. secretion

A

D. secretion

306
Q

PAH actually measures ________.
A. transport maximum
B. total renal blood flow
C. glomerular filtration rate
D. renal plasma clearance

A

B. total renal blood flow

307
Q

The presence of renin secreting tumors may cause ________.
A. hypertension
B. increased aldosterone secretion
C. increased renal sodium reabsorption
D. All of the choices are correct.

A

D. All of the choices are correct.

308
Q

An increase in extracellular H+ concentration may result in ________ extracellular K+.
A. increased
B. decreased
C. no change in

A

A. increased

309
Q

Depletion of extracellular K+ may induce ________.
A. metabolic acidosis
B. metabolic alkalosis
C. hyperventilation
D. None of the choices are correct.

A

B. metabolic alkalosis

310
Q

Since most filtered bicarbonate is reabsorbed, where does the bicarbonate come from that is used to compensate for acidosis?

A. Glutamine
B. Carbon dioxide
C. Carbonic acid
D. Glucose

A

A. Glutamine

311
Q

Uremia may cause ________.
A. acidosis
B. hyperkalemia
C. coma
D. All of the choices are correct.

A

D. All of the choices are correct.

312
Q

Which sphincter is under voluntary control?

A. External urethral sphincter
B. Internal urethral sphincter

A

A. External urethral sphincter

313
Q

What structures carry urine from the kidneys to the urinary bladder?

A. Pyramids
B. Urethras
C. Ureters
D. Calyces

A

C. Ureters

314
Q

The guarding reflex allows the urinary bladder to fill by ________.

A. inhibiting parasympathetic stimulation of the detrusor muscle

B. stimulating sympathetic nerves to the internal urethral sphincter to contract

C. responding to stretch receptors in the bladder wall

D. All of the choices are correct

A

D. All of the choices are correct

315
Q

A patient with hypertension and poorly controlled diabetes complains of frequent urination. She does not take any medications yet for these conditions. What can explain her complaint?

A. Glomerular filtration rate decreases in response to excessive stretch of the glomerular capillaries in hypertension.

B. Blood glucose increases the osmotic pressure in the tubules, causing increased urine production.

C. Renal autoregulation prevents blood glucose from entering the filtrate, but the pressure in the glomerulus increases filtrate production.

D. Sympathetic stimulation increases when blood pressure increases, which causes dilation of the afferent arterioles.

A

B. Blood glucose increases the osmotic pressure in the tubules, causing increased urine production.

316
Q

Increased sympathetic stimulation will cause ________.

A. vasoconstriction of afferent arterioles

B. decreased blood volume

C. increased GFR

D. increased urine production

A

A. vasoconstriction of afferent arterioles

317
Q

Through renal autoregulation, an increase in blood pressure will cause the afferent arteriole to ________ and therefore the GFR will ________.

A. constrict; not change
B. constrict; decrease
C. dilate; decrease
D. dilate; not change

A

A. constrict; not change

318
Q

The ________ acts as a countercurrent exchanger.

A. macula densa
B. juxtaglomerular apparatus
C. peritubular capillaries
D. vasa recta

A

D. vasa recta

319
Q

The return of molecules from the nephron tubules to the blood is called as ________.

A. reabsorption
B. filtration
C. secretion

A

A. reabsorption

320
Q

Neurohypophyseal secretion of ________ stimulates formation of concentrated urine.

A. aldosterone
B. antidiuretic hormone
C. angiotensin I
D. Renin

A

A. aldosterone

321
Q

The minimum urine volume needed to excrete metabolic wastes produced by the body is referred to as ________.

A. obligatory water loss
B. renal plasma threshold
C. renal autoregulation
D. renal transport minimum

A

A. obligatory water loss

322
Q

The reabsorption of Na+ in the proximal tubule creates an osmotic gradient for the diffusion of ________ to occur.

A. Cl-
B. H2O
C. K+
D. All of the choices are correct.

A

B. H2O

323
Q

If the rate of urine formation is 2 ml/min, the urine inulin concentration is 25 mg/ml, and the plasma inulin concentration 0.5 mg/ml, then the GFR is ________.

A. 0.04 mL/min
B. 100 mL/min
C. 6.25 mL/min
D. None of the choices are correct.

A

B. 100 mL/min

324
Q

If the glomerular filtration rate decreased, one would expect to see a corresponding ________ clearance rate.

A. increased plasma protein
B. decreased plasma glucose
C. increased plasma urea
D. decreased plasma creatinine

A

D. decreased plasma creatinine

325
Q

People with diabetes mellitus have glycosuria because ________.

A. the total renal blood flow is low
B. the plasma glucose concentration is greater than the renal plasma threshold
C. they have inadequate amounts of ADH
D. None of the choices are correct.

A

B. the plasma glucose concentration is greater than the renal plasma threshold

326
Q

Inhibition of ________ secretion from the adrenal cortex stimulates hyperkalemia.

A. angiotensin I
B. renin
C. aldosterone
D. angiotensin II

A

C. aldosterone

327
Q

Angiotensin-converting enzyme is mostly present in the capillaries of the ________.

A. adrenal medulla
B. kidneys
C. liver
D. Lungs

A

D. Lungs

328
Q

Granular cells of the juxtaglomerular apparatus respond to decreased blood volume and increased sympathetic nerve activity by secreting ________.

A. renin
B. angiotensinogen
C. inulin
D. natriuretic hormone

A

A. renin

329
Q

Neuromuscular disorders and electrocardiographic abnormalities can result from ________.

A. hypokalemia
B. excessive use of loop diuretics
C. excessive use of thiazide diuretics
D. All of the choices are correct.

A

D. All of the choices are correct.

330
Q

The ________ is a capillary network that produces a blood filtrate that enters the urinary tubules.
A. glomerulus
B. interlobar capillary
C. vasa recta
D. peritubular capillary

A

A. glomerulus

331
Q

The efferent arteriole delivers blood to the ________.
A. peritubular capillaries
B. vasa recta
C. juxtaglomerular apparatus
D. glomerulus

A

A. peritubular capillaries

332
Q

The ________ nephrons play an important role in producing concentrated urine.
A. juxtamedullary
B. cortical
C. calical
D. pelvic

A

A. juxtamedullary

333
Q

Destruction or loss of the ________ could cause albumin levels in the urine to increase.

A. parietal layer of the capsule
B. capillary fenestrae
C. slit diaphragm
D. glomerular basement membrane

A

C. slit diaphragm

334
Q

Which of the following is a layer of the renal corpuscle?
A. Glomerular visceral epithelium
B. Glomerular parietal epithelium
C. Glomerular endothelium
D. All of the choices are correct.

A

D. All of the choices are correct.

335
Q

The concentration of renal filtrate is highest in the ________.
A. proximal convoluted tubule
B. distal convoluted tubule
C. base of the nephron loop
D. cortical portion of the collecting duct

A

C. base of the nephron loop

336
Q

Which of the following is a nonsalt molecule that contributes to the hypertonicity of the interstitial fluid of the renal tubules?

A. Creatinine
B. Protein
C. Xenobiotic molecule
D. Urea

A

D. Urea

337
Q

The ________ is the saturation point of renal carrier molecules.

A. transport maximum
B. GFR
C. renal plasma threshold
D. renal plasma clearance

A

A. transport maximum

338
Q

Where are OATs located?

A. Basolateral membrane of proximal tubule cells
B. Basolateral membrane of distal tubule cells
C. Apical surface of collecting duct cells
D. Apical surface of distal tubule cells

A

A. Basolateral membrane of proximal tubule cells

339
Q

The renal plasma clearance of a substance that is filtered but not reabsorbed or secreted is ________ the GFR.
A. less than
B. greater than
C. equal to

A

C. equal to

340
Q

Inhibiting the actions of ________ would impair the ability of the kidney to regulate pH of the blood.
A. ADH
B. carbonic anhydrase
C. inulin
D. aldosterone

A

B. carbonic anhydrase

341
Q

Due to the action of aldosterone, water is reabsorbed because ________.
A. aldosterone opens aquaporins
B. sodium is reabsorbed into peritubular blood
C. potassium is reabsorbed into peritubular blood
D. sodium is excreted in the urine

A

B. sodium is reabsorbed into peritubular blood

342
Q

Changes in blood volume are detected by ________.
A. stretch receptors in the atria
B. osmoreceptors in the medulla
C. osmoreceptors in hypothalamus
D. cells in the proximal convoluted tubule

A

A. stretch receptors in the atria

343
Q

Hemorrhage would induce ________.
A. increased renin secretion
B. increased atrial natriuretic peptide secretion
C. decreased ADH secretion
D. All of the choices are correct

A

A. increased renin secretion

344
Q

An autoimmune disease that attacks the basement membranes of the glomerular capillaries could result in ________.

A. increased tubular osmolality

B. increased permeability to sodium in the distal convoluted tubules

C. increased plasma colloid osmotic pressure

D. increased permeability to water aquaporins in the collecting duct

A

A. increased tubular osmolality

345
Q

The renal pelvis receives urine from the ________.

A. major calyces
B. ureters
C. minor calyces
D. renal pyramids

A

A. major calyces

346
Q

Damage to ________ would not impair micturition.

A. parasympathetic neurons from the lumbar region
B. sympathetic neurons from the sacral region
C. pudendal somatic motor neurons
D. the detrusor muscle

A

A. parasympathetic neurons from the lumbar region

347
Q

Which of the following is an autosomal dominant inherited condition in which progressive renal failure develops due to expanded portions of the renal tubule?
A. Polycystic kidney disease
B. Renal calculus
C. Pyelonephritis
D. Glomerulonephritis

A

A. Polycystic kidney disease

348
Q

Hypotension would induce ________ afferent arterioles.
A. vasodilation of
B. no change in
C. vasoconstriction of

A

A. vasodilation of

349
Q

What is the order that filtrate must pass through to the glomerular capsule?

A. Glomerular basement membrane, capillary fenestrae, podocytes of visceral glomerular epithelium

B. Capillary fenestrae, podocytes of visceral glomerular epithelium, glomerular basement membrane

C. Capillary fenestrae, glomerular basement membrane, podocytes of visceral glomerular epithelium

D. Podocytes of the glomerular epithelium, glomerular basement membrane, capillary fenestrae

A

C. Capillary fenestrae, glomerular basement membrane, podocytes of visceral glomerular epithelium

350
Q

What is present in the vasa recta to remove water for the interstitial fluid of the renal medulla?

A. Aquaporins
B. ADH receptors
C. Na+/K+ pump
D. Urea transporters

A

A. Aquaporins

351
Q

The ________ limb of the nephron loop is impermeable to salt.

A. ascending
B. descending
C. both limbs
D. neither limb

A

B. descending

352
Q

The majority of reabsorption occurs in the ________.

A. proximal convoluted tubule
B. collecting duct
C. nephron loop
D. distal convoluted tubule

A

A. proximal convoluted tubule

353
Q

People with diabetes mellitus have polyuria because they ________.

A. have inadequate amounts of ADH
B. have greater oncotic pressure in the kidney tubules
C. drink a lot of water
D. have inadequate amounts of aldosterone

A

B. have greater oncotic pressure in the kidney tubules

354
Q

Excessive aldosterone secretion would induce ________.
A.decreased plasma potassium
B.decreased plasma sodium
C.decreased plasma chloride
D.no change in plasma ion balance

A

A.decreased plasma potassium

355
Q

Aldosterone secretion from the adrenal cortex is stimulated by a(n) ________ in blood Na+ or a(n) ________ in blood K+.

A. decrease; decrease
B. increase; increase
C. increase; decrease
D. decrease; increase

A

D. decrease; increase

356
Q

Why is the urinary albumin excretion rate an indication of renal damage from diabetes or hypertension?

A. Both diabetes and hypertension result in excess retention of albumin by the kidneys.
B. Diabetes and hypertension may damage the glomerulus, causing excess albumin loss during filtration.
C. Diabetes results in increased plasma albumin concentrations, which damages the glomerulus.
D. Albumin in the urine is a sign of end-stage renal failure.

A

B. Diabetes and hypertension may damage the glomerulus, causing excess albumin loss during filtration.

357
Q

The ________ is the functional unit of the kidney.

A. calyx
B. nephron
C. medulla
D. neuron

A

B. nephron

358
Q

What type of cells form the third filtration barrier?

A. Capillary simple squamous epithelial cells
B. Ciliated cuboidal epithelial cells
C. Glomerulocytes
D. Podocytes

A

D. Podocytes

359
Q

Which of the following conditions could cause diabetes insipidus?

A. Exceeding transport maximum levels of glucose in the renal tubule
B. Pituitary tumor
C. Excess production of ADH from the pituitary gland
D. Autoimmune disorder of the pancreas

A

B. Pituitary tumor

360
Q

The obligatory water loss needed to excrete metabolic wastes is ________ per day.

A. 300 mL
B. 800 mL
C. 400 mL
D. 600 mL

A

C. 400 mL

361
Q

The effectiveness of some antibiotics, such as penicillin, is enhanced by preventing their excretion by use of ________.

A. xenobiotics
B. organic anion transporters
C. toxiporins
D. Probeneci

A

D. Probeneci

362
Q

Renal clearance of PAH ________ glomerular filtration rate.

A. Is less than
B. Is equal to
C. Exceeds

A

C. Exceeds

363
Q

The minimum plasma concentration of a substance that results in the excretion of that substance in the urine is the ________.

A. transport maximum
B. renal plasma threshold
C. renal plasma clearance
D. GFR

A

B. renal plasma threshold

364
Q

The main region of aldosterone action is the ________.

A. medullary collecting duct
B. proximal convoluted tubule
C. distal convoluted tubule
D. cortical collecting duct

A

D. cortical collecting duct

365
Q

Bicarbonate must be converted to ________ to move into tubule cells and reduce blood pH.

A. carbonic anhydrase
B. carbonic acid
C. carbon dioxide
D. None of the choices are correct.

A

C. carbon dioxide

366
Q

Urinalysis on a patient reveals a number of abnormalities. Choose the abnormal finding that best indicates damage to the filtration barriers in the glomerulus may have occurred.

A. Hypercalcemia
B. Polyuria
C. Increased creatinine
D. Hematuria

A

D. Hematuria

367
Q

Antidiuretic hormone _________.

A. stimulates water reabsorption by the kidney

B. induces fusion of aquaporin containing vesicles with the plasma membrane

C. is secreted in response to dehydration

D. All of the choices are correct

A

D. All of the choices are correct

368
Q

Tubular ultrafiltrate enters the collecting duct from the ________.

A. Distal convoluted tubule
B. Glomerulus
C. Proximal convoluted tubule
D. Nephron loop

A

A. Distal convoluted tubule

369
Q

What creates a Na+ concentration gradient in the proximal convoluted tubule?

A. Diffusion of Cl-
B. Osmosis of water
C. Na+/K+ pump
D. Diffusion of Na+

A

C. Na+/K+ pump

370
Q

An inadequate dietary intake of NaCl will cause ________.

A. a rise in renal blood flow
B. increased renin secretion
C. decreased aldosterone secretion
D. a rise in blood volume

A

B. increased renin secretion

371
Q

Increased stimulation of the renin-angiotensin-aldosterone system will trigger which of the following to occur?

A. Increased transport of sodium into the urine

B. Decreased plasma osmolality

C. Increased tubular osmolality

D. Increased transport of potassium into the urine

A

D. Increased transport of potassium into the urine

372
Q

Loop diuretics such as Lasix ________.

A. inhibit the actions of ADH

B. inhibit salt transport in the first section of the distal tubule

C. inhibit active transport of salt out of the ascending nephron loop

D. add extra solutes to the filtrate

A

C. inhibit active transport of salt out of the ascending nephron loop

373
Q

Which of the following is the correct order of the nephron tubule structures?

A. Nephron loop, Bowman’s capsule, proximal convoluted tubule, distal convoluted tubule

B. Bowman’s capsule, proximal convoluted tubule, nephron loop, distal convoluted tubule

C. Proximal convoluted tubule, Bowman’s capsule, distal convoluted tubule, nephron loop

D. Distal convoluted tubule, nephron loop, proximal convoluted tubule, B

A

B. Bowman’s capsule, proximal convoluted tubule, nephron loop, distal convoluted tubule

374
Q

The average glomerular filtration rate is equal to ________.

A. 180 L/day
B. 100 mmHg/hour
C. 5.5 L/min
D. 8 mg/hour

A

A. 180 L/day

375
Q

Na+ reabsorption is a(n) ________ process, while Cl- reabsorption is a(n) ________ process.

A. passive; active
B. active; active
C. active; passive
D. passive; passive

A

C. active; passive

376
Q

Renal insufficiency ________.

A. stimulates metabolic alkalosis
B. stimulates hypokalemia
C. may occur as a result of arteriosclerosis
D. often results from dialysis

A

C. may occur as a result of arteriosclerosis

377
Q

Which structure is an arteriole capillary bed drained by an arteriole instead of a venule?

A. Vasa recta
B. Glomerulus
C. Peritubular capillaries
D. Juxtaglomerular apparatus

A

B. Glomerulus

378
Q

Glomerular filtration would be decreased by ________.

A. increased mean arterial pressure

B. increased glomerular filtrate protein concentration

C. increased plasma protein concentration

D. decreased glomerular hydrostatic pressure

A

D. decreased glomerular hydrostatic pressure

379
Q

Inhibition of the functions of the descending limb of the nephron loop would stimulate ________.

A. decreased Na+ reabsorption

B. decreased water reabsorption

C. increased Na+ reabsorption

D. increased water reabsorption

A

B. decreased water reabsorption

380
Q

Diuretics ________.

A. primarily act on the distal convoluted tubule
B. induce hypertension
C. decrease urine production
D. primarily act on the proximal convoluted tubule

A

A. primarily act on the distal convoluted tubule

381
Q

The conversion of angiotensin I into angiotensin II occurs primarily in the ________.

A. liver
B. kidneys
C. adrenal cortex
D. Lungs

A

D. Lungs

382
Q

Shock-wave lithotripsy is used to treat ________.

A. Glomerulonephritis
B. Acute renal failure
C. Renal calculi
D. Polycystic kidney disease

A

C. Renal calculi

383
Q

The ability of the kidneys to remove molecules from the blood plasma by excreting them in the urine known as ________.

A. Reabsorption
B. Glomerular filtration
C. Renal clearance
D. Micturition

A

C. Renal clearance

384
Q

Urea transport in the collecting duct is a(n) ____ process.

A. Active
B. Endocytosis
C. Passive
D. Exocytosis

A

C. Passive